Angular Momentum and Rotations

Motivating Questions

  • What comes to mind when thinking about angular momentum?
    • Rotations
    • \(\vec{L}=\vec{r}\times \vec{p}\)
    • A quantity that is conserved in systems (with no external torque) with invariance under rotations
  1. Geometrical Rotations
  2. Rotations of state vectors \(|\Psi\rangle\Rightarrow|\Psi'\rangle=\mathcal{D}(R)|\Psi\rangle\)
  3. \(\mathcal{D}(R)\Leftrightarrow J\), the generalized angular momentum

Geometrical Rotations

Rotate about \(z\) axis by \(\varphi\), we can use an active rotation (xyz coordinate system remain the same), \(\vec{r}\to\vec{r}'\) with an angle in the xy-plane of \(\varphi\). passive rotation (xyz coordinate system changes), \(\vec{r}\to\vec{r}'\) with the coordinate \(x',y'\) have an angle \(\varphi\) between them (\(-\varphi\) between \(\vec{r}\) and \(\vec{r}'\)).

We will use active rotations.

We accomplish this with \(\vec{r}' = R\vec{r}\) with \(R\) being a 3x3 orthogonal matrix. Orthogonal \((R^TR=RR^T=\mathbb{I})\) is due to \(|\vec{r}'|=|\vec{r}|\).

Then, \(R(\varphi)=\begin{pmatrix} \cos\varphi & -\sin\varphi & 0 \\ \sin\varphi & \cos\varphi & 0 \\ 0 & 0 & 1 \end{pmatrix}\).

\(R_x(\varphi)=\begin{pmatrix} 1 & 0 & 0 \\ 0 & \cos\varphi & -\sin\varphi \\ 0 & \sin\varphi & \cos\varphi \end{pmatrix}\).

\(R_y(\varphi)=\begin{pmatrix} \cos\varphi & 0 & \sin\varphi \\ 0 & 1 & 0 \\ -\sin\varphi & 0 & \cos\varphi \end{pmatrix}\).

Recall that rotations are non-commutative \(R_i(\varphi)R_j(\varphi)\neq R_j(\varphi')R_i(\varphi)\).

For a small angle \([R_x(\varepsilon),R_y(\varepsilon)]=\begin{pmatrix} 0 & -\varepsilon^2 & 0 \\ \varepsilon^2 & 0 & 0 \\ 0 & 0 & 0 \end{pmatrix}=R_z(\varepsilon^2)-\mathbb{I}\to0\).

\(R_i(\varphi+\varepsilon) = R_i(\varphi)R_i(\varepsilon) = R_i(\varepsilon)R_i(\varphi)\).

The set of rotations R constitutes a group

An abstract group is a set of elements \(G\) and a binary operation \(\cdot\) such that for \(a,b\in G\) satisfies:

  1. \(a\cdot b\in G\)
  2. \(\exists e.a\cdot e = e\cdot a= a\)
  3. \(\exists e.\forall a.\exists a^{-1}.a\cdot a^{-1} = e\)
  4. The binary operation is associative: \((a\cdot b)\cdot c=a\cdot(b\cdot c)\)

A group can be finite if the set is finite and infinite if there are infinitely many. The infinite case can be countably infinite or uncountably infinite.

\(\mathbb{Q}\) is denumerable since it gives an injection to \((\mathbb{N},\mathbb{Z})\) which is equivalent to \(\mathbb{N}\)?

  • Examples
    • \(\mathbb{Z}\) under multiplication? No, the inverse element does not exist!
    • \(\mathbb{Z}\) under addition? Yes
    • Rotations Yes. \(R_n(\varphi)R_m(\varphi')\in\) rotations. \(R^TR=RR^T=R_n(\varphi)R_m(\varphi')R_m(\varphi')R_n(\varphi)=\mathbb{I}=R_o(0)\). \(R^T=R^{-1}\). \((R_n(\varphi)R_m(\varphi'))R_o(\varphi'')=R_n(\varphi)(R_m(\varphi')R_o(\varphi''))\).

Rotation Groups

\(O(3)\) is the group of all 3x3 orthogonal matrices.

\(\det R=+1\) implies rotations. \(\det R=-1\) implies parity.

\(SO(3)\) gives the rotations in 3 dimensions.

Abelian groups are commutative. Non-abelian groups are non-commutative (SO(3)).

Rotations in State Space

\(|\Psi\rangle \Rightarrow |\Psi'\rangle=\mathcal{D}(R)|\Psi\rangle\). Where \(\mathcal{D}\) stands for Drehung rotation.

\(\mathcal{D}\mathcal{D}^\dagger\). Remember generators for time, \(U_\varepsilon = 1-iG\varepsilon\) with \(G=\frac{H}{\hbar}\) \(\varepsilon=dt\). For space, \(G=\frac{p_x}{\hbar}\), \(\varepsilon=dx\). For rotations, \(G=\frac{J_z}{\hbar}=\frac{\vec{J}\cdot\vec{n}}{\hbar}\) \(\varepsilon=d\varphi\). For finite rotations, \(\mathcal{D}(\vec{n},\varphi)=\exp\left(\frac{-i}{\hbar}\vec{J}\cdot\vec{n}d\varphi\right)\)

Remember, \(A' = UAU^\dagger\). So, \(A' = \mathcal{D}(R)A\mathcal{D}^\dagger(R)\).

Remember, if a generator commutes with the observable, then \(A\) is conserved under the transformation. I.e. \(A'=\mathcal{D}(R)A\mathcal{D}^\dagger(R) \approx A-\frac{i}{\hbar}d\varphi[\vec{J}\cdot\vec{n},A]\). If this is \(A\) then \(A\) is conserved under the transformation and it is also called a scalar.

\(A'= A + \left[-\frac{i}{\hbar}(\vec{J}\cdot\vec{n})d\varphi,A\right] + \frac{1}{2!} \left[-\frac{i}{\hbar}(\vec{J}\cdot\vec{n})d\varphi,\left[-\frac{i}{\hbar}(\vec{J}\cdot\vec{n})d\varphi,A\right]\right] + \cdots\).

Let us consider the Hamiltonian. Let \(|\Psi(t_0)\rangle\) be transformed by \(\mathcal{D}(R)\) to \(|\Psi'(t_0)\rangle=\mathcal{D}(R)|\Psi(t_0)\rangle\) then be transformed by \(|\Psi'(t_0+dt)\rangle\).

Consider the opposite order, \(|\Psi(t_0)\rangle\) be time transformed \(|\Psi(t_0+dt)\rangle\) then rotated \(|\Psi''(t_0+dt)\rangle\).

\(\Psi'(t_0+dt)\rangle = |\Psi'(t_0)\rangle + dt\left.\frac{d|\Psi'\rangle}{dt}\right|_{t_0} = |\Psi'(t_0)\rangle + \left.\frac{dt}{i\hbar}\hat{H}|\Psi'\rangle\right|_{t_0} = \mathcal{D}(R)|\Psi(t_0)\rangle + \frac{dt}{i\hbar}\hat{H}\mathcal{D}(R)|\Psi(t_0)\rangle\).

\(|\Psi(t+dt)\rangle = |\Psi(t_0)\rangle + \frac{dt}{i\hbar}\hat{H}|\Psi(t_0)\rangle\). So, \(\mathcal{D}(R)|\Psi(t+dt)\rangle = \mathcal{D}(R)|\Psi(t_0)\rangle + \frac{dt}{i\hbar}\mathcal{D}(R)\hat{H}|\Psi(t_0)\rangle\). If we expect the states to be the same, then the Hamiltonian commutes with the rotation operator. If the states are different, then the Hamiltonian does not commute.

So, \([\hat{H},\vec{J}\cdot\vec{n}]=0\Rightarrow \hat{H}\) is a scalar.

Recall: \(\frac{d\langle A\rangle}{dt} = \langle\frac{\partial A}{\partial t}\rangle + \frac{1}{i\hbar}\langle[A,H]\rangle\).

If \(A=\vec{J}\) then \(\frac{d\langle J\rangle}{dt}=\frac{1}{i\hbar}\langle[\vec{J},H]\rangle=0\) implies that \(\vec{J}\) is a constant of the motion.

Spin

Rotations in Spin Space

\(\mathcal{D}^{(s)}(\vec{n},\varphi) = \exp\left(-\frac{i}{\hbar}\varphi\vec{S}\cdot\vec{n}\right)\)

For \(s=\frac{1}{2}\), \(\vec{S}=\frac{\hbar}{2}\vec{\sigma}\), \(\mathcal{D}^{(s)}(\vec{n},\varphi) = \exp\left(-i\frac{\varphi}{2}\hat{\vec{\sigma}}\cdot\vec{n}\right) \approx \mathbb{I}\cos\frac{\varphi}{2}-i(\vec{\sigma}\cdot\vec{n})\sin\frac{\varphi}{2}\). (Not sure if actually approximate or exact). \(\hat{\vec{\sigma}}\cdot\vec{n}=\sigma_xn_x+\sigma_yn_y+\sigma_zn_z=\begin{pmatrix} n_z & n_x-in_y \\ n_x+in_y & -n_z \end{pmatrix}\). \(\exp(\cdots)=\begin{pmatrix} \cos\frac{\varphi}{2}-in_z\sin\frac{\varphi}{2} & -i\sin\frac{\varphi}{2}(n_x-in_y) \\ -i\sin\frac{\varphi}{2}(n_x+in_y) & \cos\frac{\varphi}{2}+in_z\sin\frac{\varphi}{2} \end{pmatrix}\)

For \(\varphi=2\pi\), \(\mathcal{D}^{(s)} = -\mathbb{I}\). So \(\mathcal{D}^{(s)}|\alpha\rangle=-|\alpha\rangle\). For \(\varphi=4\pi\), \(\mathcal{D}^{(s)} = \mathbb{I}\).

Thus, we get SU(2).

Representations of the Rotation Operator

An arbitrary rotation of a rigid body can be accomplished in 3 rotations, Euler angles \(\alpha,\beta,\gamma\). \(R(\alpha,\beta,\gamma)=R_{z'}(\gamma)R_{y'}(\beta)R_z(\alpha)\) From Goldstein, \(R(\alpha,\beta,\gamma) = R_z(\alpha)R_y(\beta)R_z(\gamma)\)

For spin space, where \(s=\frac{1}{2}\), \(\mathcal{D}(\alpha,\beta\gamma) = \mathcal{D}_z(\alpha) \mathcal{D}_y(\beta) \mathcal{D}_y(\gamma)=\exp\frac{-i\sigma_z\alpha}{2}\exp\frac{-i\sigma_y\beta}{2}\exp\frac{-i\sigma_z\gamma}{2} = \begin{pmatrix} \exp\frac{-i\alpha}{2} & 0 \\ 0 & \exp\frac{i\alpha}{2} \end{pmatrix}\cdots = \begin{pmatrix} \exp\left(\frac{-i(\alpha+\gamma)}{2}\right)\cos\frac{\beta}{2} & \exp\left(\frac{-i(\alpha-\gamma)}{2}\right)\sin\frac{\beta}{2} \\ \exp\left(\frac{i(\alpha-\gamma)}{2}\right)\sin\frac{\beta}{2} & \exp\left(\frac{i(\alpha+\gamma)}{2}\right)\cos\frac{\beta}{2} \\ \end{pmatrix}\) called the \(j=\frac{1}{2}\) irreducible representation of the rotation operator.

\(\mathcal{D}_{m_s'm_s}^{(1/2)}(\alpha,\beta,\gamma)=\langle \frac{1}{2}m_s'|\exp(-iS_z\alpha/\hbar)\exp(-iS_y\beta/\hbar)\exp(-iS_z\gamma/\hbar)|\frac{1}{2}m_s\rangle\)

By replacing \(S_i\) with \(J_i\) we get a more general expression, \(\mathcal{D}_{m_j'm_j}^{(j)}=\langle jm_j'|\exp\left(\frac{-i}{\hbar}J_z\alpha\right)\exp\left(\frac{-i}{\hbar}J_y\beta\right)\exp\left(\frac{-i}{\hbar}J_z\gamma\right)|jm_j\rangle\).

Question:

  • \(\mathcal{D}|j,m\rangle \Leftrightarrow c|j,m'\rangle\)
  • Why do we not go to some \(j'\)?

Generalized rotation operator matrix element, \(\mathcal{D}_{m'm}^{(j)}(R) = \langle jm'|\exp\left(-\frac{i}{\hbar}\vec{J}\cdot\vec{n}\varphi\right)|jm\rangle\)

Remember that \([\vec{J}^2,\vec{J}] = 0\) so \([\vec{J}^2,\mathcal{D}(R)] = 0\). Thus, the \(\vec{J}^2\) eigenvalue is unchanged under \(\mathcal{D}(R)\).

I.e. \(\vec{J}^2\mathcal{D}(R)|jm\rangle = \vec{J}^2|\Psi\rangle\), \(\mathcal{D}(R)\vec{J}^2|jm\rangle = \mathcal{D}(R)\hbar^2j(j+1)|jm\rangle\) A zero commutator implies that \(|\Psi\rangle = \mathcal{D}(R)|jm\rangle\). Thus, \(m\) may change but not \(j\).

Inserting Identity, \(\mathcal{D}(R)|jm\rangle = \sum_{m'}|jm'\rangle\langle jm'|\mathcal{D}(R)|jm\rangle = \sum_{m'}\mathcal{D}_{m'm}^{(j)}(R)|jm'\rangle\). This gives the weights for each of the eigenstates. I.e. the probabilities to end up in the particular state.

Irreducible: The general rotation matrix has the sub rotation matrices on the diagonals, but due to the zeroes on the diagonals separating them, they cannot go between the \(j\) bases. I.e. there is no crossover.

\(\mathcal{D}_{m'm}^{(j)}(R)\) is a \((2j+1)\times(2j+1)\) matrix, \((2j+1)-\) dim irreducible representation of \(\mathcal{D}(R)\) implies that any \(|jm\rangle\) can be rotated by \(\mathcal{D}(R)|jm\rangle\) to yield \(|jm'\rangle\) which then span the entire \(\mathcal{E}_j\).

Returning to the matrix elements:

\begin{align*} \mathcal{D}_{m_j'm_j}^{(j)}(\alpha,\beta,\gamma) & =\langle jm_j'|\exp\left(-\frac{i}{\hbar}J_z\alpha\right)\exp\left(-\frac{i}{\hbar}J_y\beta\right)\exp\left(-\frac{i}{\hbar}J_z\gamma\right)|jm_j\rangle \\ & =\exp(-im'\alpha)\langle jm_j'|\exp\left(-\frac{i}{\hbar}J_y\beta\right)\exp(-im\gamma)|jm_j\rangle \\ &= \exp(-im'\alpha+m\gamma)d_{m'm}^{(j)} \end{align*}

Using the ladder operators, \(J_y = \frac{J_+-J_-}{2i}\), \(J_{\pm}=\hbar\sqrt{j(j+1)-m(m\pm 1)}|jm\pm1\rangle\), \(\langle 1m'|J_y|1m\rangle\Rightarrow \begin{pmatrix} 0 & -i\sqrt{2} & 0 \\ i\sqrt{2} & 0 & -i\sqrt{2} \\ 0 & i\sqrt{2} & 0 \end{pmatrix}\). Taylor expanding the exponential,

\begin{align*} \exp\left(-\frac{i}{\hbar}J_y\beta\right)&=\mathbb{I}-\frac{J_y^2}{\hbar^2}(1-\cos\beta)-i\frac{J_y}{\hbar}\sin\beta. \end{align*}

So,

\begin{align*} j = 1\Rightarrow d^{(1)}(\beta) &= \begin{pmatrix} \frac{1+\cos\beta}{2} & -\frac{1}{\sqrt{2}}\sin\beta & \frac{1}{2}(1-\cos\beta) \\ \frac{\sqrt{2}}{2}\sin\beta & \cos\beta & \frac{-\sqrt{2}}{2}\sin\beta \\ \frac{1}{2}(1-\cos\beta) & \frac{\sqrt{2}}{2}\sin\beta & \frac{1}{2}(1+\cos\beta) \end{pmatrix}. \end{align*}

Since the overall rotation matrix is unitary, we expect the sub matrix d to be orthogonal due to the fact that it is real. \(d^{(j)}(0) = \mathbb{I}\). Sum of the squares of the rows or columns is 1.

Spherical Harmonics

The connection between the Spherical Harmonics and the rotation matrices.

Recall: \(\mathcal{D}(\alpha,\beta,\gamma)|jm\rangle = \sum_{m'=-j}^j \mathcal{D}_{m'm}^{(j)}(\alpha,\beta,\gamma)|jm'\rangle = \sum_{m'=j}^j\exp\left(-i(m'\alpha+m\gamma)\right)d_{m'm}^{(j)}(\beta)|jm'\rangle\)

Wigner Formula

\(d_{m'm}^{(j)} = \sum_{k}(-1)^{k+m'-m}\frac{\sqrt{(j+m)!(j-m)!(j+m')!(j-m)!}} {(j-m'-k)!(j+m-k)!(k+m'-m)!k!} \left(\cos\frac{\beta}{2}\right)^{2j+m-m'-2k} \left(\sin\frac{\beta}{2}\right)^{m'-m+2k}\) The \(k\) s are such that the factorials in the denominator are positive.

\(|\ell m\rangle \doteq Y_\ell^m(\Omega)\). \(\langle\vec{n}|\ell m\rangle = Y_\ell^m(\Omega)\).

Start with some \(|\vec{z}\rangle\) and rotate it on \(|\vec{n}\rangle\). Hence, \(\mathcal{D}(R)|\vec{z}\rangle = |\vec{n}\rangle = \sum_m\mathcal{D}(R)|\ell m\rangle\langle \ell m|\vec{z}\rangle\).

Multiply by \(\langle \ell m'|\) so, \(\langle \ell m'|\vec{n}\rangle = \sum_m\langle \ell m'|\mathcal{D}(R)|\ell m\rangle\langle \ell m|\vec{z}\rangle\). Thus, \(Y_\ell^{m*}(\Omega) = \sum_m \mathcal{D}_{m'm}^{(\ell)}(\alpha,\beta,\gamma)\langle \ell m|\vec{z}\rangle\).

Ansatz: \(\varphi = \alpha+\gamma\), \(\theta = \beta\).

Since rotation about \(z\) leaves \(\vec{z}\) unchanged, we can set \(\gamma=0\) without loss of generality. Then, \(\theta = \beta\). So, \(\varphi=\alpha\). Thus, \(Y_\ell^{m'* }(\Omega) = \sum_m \mathcal{D}_z(\theta)\mathcal{D}_y(\varphi)\langle \ell m|\vec{z}\rangle\). \(\langle \ell m|\vec{z}\rangle = Y_{\ell}^{m*}(\theta=0,\varphi) = Y_\ell^{0*}(0)\delta_{m,0} = \sqrt{\frac{2\ell+1}{4\pi}}P_\ell(\cos 0)\delta_{m,0} = \sqrt{\frac{2\ell+1}{4\pi}}\delta_{m,0}\)

Inserting this back into the sum, \(Y_\ell^{m'*}(\theta,\varphi) = \mathcal{D}_{m'0}^{(\ell)}(\alpha=\varphi,\beta=\theta,\gamma=0)\sqrt{\frac{2\ell+1}{4\pi}}\). So, \(\mathcal{D}_{m'0}^{(\ell)}(\alpha,\beta,\gamma=0)=\left.\sqrt{\frac{4\pi}{2\ell+1}}Y_{\ell}^{m'*}(\theta,\varphi)\right|_{\alpha=\varphi,\beta=\theta}\).

\(\mathcal{D}_{m'm}^{(j)} = \exp(-im'\alpha+m\gamma)d_{m'm}^{(j)}(\beta)\).

For \(j=1\). Recall we set \(\gamma=0\). Then, \(\mathcal{D}_{m'm}^{(1)}=\exp(-im'\alpha)\begin{pmatrix} \frac{1}{2}(1+\cos\beta) & \frac{-\sin\beta}{\sqrt{2}} & \frac{1}{2}(1-\cos\beta) \\ \cdots & \cos\beta & \cdots \\ \cdots & \frac{\sin\beta}{\sqrt{2}} & \cdots \end{pmatrix}\)

The rest of the matrix is on the worksheet.

The middle column is then our spherical harmonics.

So, \(\mathcal{D}_{m'0}^{(1)}=\exp(-im'\varphi)\begin{pmatrix} \frac{-\sin\theta}{\sqrt{2}} \\ \cos\theta \\ \frac{\sin\theta}{\sqrt{2}} \end{pmatrix} = \begin{pmatrix} \exp(-i\varphi)\frac{-\sin\theta}{\sqrt{2}} \\ \cos\theta \\ \exp(i\varphi)\frac{\sin\theta}{\sqrt{2}} \end{pmatrix} = \sqrt{\frac{4\pi}{3}}\begin{pmatrix} Y_1^{1*} \\ Y_1^{0*} \\ Y_1^{-1*} \end{pmatrix}\).

If \(m'=0\), then, \(\mathcal{D}_{00}^{(\ell)}(\varphi,\theta,0) = d_{00}^{(\ell)}(\theta) = \sqrt{\frac{4\pi}{2\ell+1}}Y_\ell^{0*}(\theta) = P_\ell(\cos\theta)\).

Addition of Angular Momenta

Spectroscopic Notation

Rotation Matrices for Coupling Two Angular Momentum

Recall: \(D(R) = D_z(\alpha)D_y(\beta)D_z(\gamma)\), \(d_{m'm}^{(j)}(\beta) = \langle jm'|\exp(-i\beta J_y/\hbar)|jm\rangle\).

How does \(d_{m'm}^{(j)}\Leftrightarrow d_{m'm}^{(j_1)}(\beta),d_{m'm}^{(j_2)}\).

\(d_{m'm}^{(j)} = \langle jm'|\exp(-i\beta J_y/\hbar)|jm\rangle = \sum_{m_1m_2}\sum_{m_1'm_2'}\langle jm'|j_1j_2m_1m_2\rangle\langle j_1j_2m_1m_2|\exp(-i\beta(J_{y_1}+J_{y_2})/\hbar)|j_1j_2m_1'm_2'\rangle\langle j_1j_2m_1'm_2'|jm\rangle = \sum_{m_1m_2}\sum_{m_1'm_2'}\langle jm'|j_1j_2m_1m_2\rangle\langle j_1j_2m_1'm_2'|jm\rangle\langle j_1j_2m_1m_2|\exp(-i\beta J_{y_1}/\hbar)|j_1j_2m_1'm_2'\rangle\langle j_1j_2m_1m_2|\exp(-i\beta J_{y_2}/\hbar)|j_1j_2m_1'm_2'\rangle = \sum_{m_1m_2}\sum_{m_1'm_2'}\langle jm'|j_1j_2m_1m_2\rangle\langle j_1j_2m_1'm_2'|jm\rangle d_{m_1'm_1}^{(j_1)}(\beta)d_{m_2'm_2}^{(j_2)}(\beta)\).

Inverting the expression, \(d_{m_1'm_1}^{(j_1)}(\beta)d_{m_2'm_2}^{(j_2)}(\beta) = \sum_{j=|j_1-j_2|}^{j_1+j_2}\sum_{mm'}\langle j_1j_2m_1m_2|jm\rangle\langle j_1j_2m_1'm_2'|jm'\rangle d_{m'm}^{(j)}(\beta)\).

We can replace the small \(d(\beta)\) with \(\mathcal{D}(\alpha,\beta,\gamma)\) for the same relationship.

Note that this is a reducible representation for \(\mathcal{D}^{(j_1)}\otimes\mathcal{D}^{(j_2)}\) since there are many different \(j\) s. We get a direct sum of irreducible: \(\mathcal{D}^{(j_1+j_2)}\oplus\cdots\oplus\mathcal{D}^{|j_1-j_2|}\)

Recall: \(\exp(A+B) = \exp(A)\exp(B)\) iff they commute.

Spherical Harmonics

\(j_1=\ell_1,j_2=\ell_2,m=0\). Recall: \(\mathcal{D}_{m'0}^{(\ell)}(\alpha,\beta,\gamma) = \sqrt{\frac{4\pi}{2\ell+1}}Y_{\ell}^{m' *}(\theta,\varphi)\).

\(\mathcal{D}_{m_1'0}^{(\ell_1)}\mathcal{D}_{m_2'0}^{(\ell_2)} = \sum_{\ell,m'}\langle \ell_1\ell_200|\ell0\rangle\langle\ell_1\ell_2m_1'm_2'|\ell m'\rangle\mathcal{D}_{m'0}^{(\ell)}\) \(Y_{\ell_1}^{m_1'}Y_{\ell_2}^{m_2} = \sum_{ell,m'}\sqrt{\frac{(2\ell_1+1)(2\ell_2+1)}{4\pi(2\ell+1)}}\langle\ell_1\ell_200|\ell0\rangle\langle \ell_1\ell_2m_1'm_2'|\ell m'\rangle Y_\ell^{m'}\)

Applying \(Y_\ell^{m*}\) and integrating over a solid angle gives an orthogonality relationship, \(\int Y_\ell^{m*}Y_{\ell_1}^{m_1}Y_{\ell_2}^{m_2}\:d\Omega = \sqrt{\frac{(2\ell_1+1)(2\ell_2+1)}{4\pi(2\ell+1)}}\langle\ell_1\ell_200|\ell0\rangle\langle\ell_1\ell_2m_2m_2|\ell m\rangle\).

Tangent

\(|\ell_2m_2\rangle \to |\ell m\rangle\), \(\mathcal{P}\sim|\langle\ell m|H_{int}|\ell_2m_2\rangle|^2\), \(H_{int} = -e\vec{r}\cdot\vec{E} = -eE(r\cos\theta) = -\alpha eE Y_{1}^0\). So, \(\int Y_{\ell}^{m*}Y_{1}^{0}Y_{\ell_2}^{m_2}d\Omega\)

Aside

Supplemental Paper

For two sources with different phases and energies, \(|E_1\exp(i\varphi_1) + E_2\exp(i\varphi_2)|^2\)

\(|\alpha,t=0\rangle = \frac{1}{\sqrt{2}}(|+\rangle + |-\rangle)\) time translated gives \(|\alpha,t\rangle = \exp\left(\frac{-i}{\hbar}Ht\right)|\alpha,0\rangle\)

Two pathways possible.

Path 1: \(H=H_0=\frac{p^2}{2m}\)

Path 2: \(H=H_0-\vec{\mu}\cdot\vec{B} = H_0 -\frac{g_ne}{m_nc}S_zB\)

Let \(\omega = \frac{g_n e}{m_n c}B\). So, \(|\alpha,t\rangle = \exp\left(\frac{i}{\hbar}S_z\omega t\right)|\alpha,0\rangle\) On our initial state, \(|\alpha,t\rangle = \frac{1}{\sqrt{2}}\exp\frac{i\omega t}{2}|+\rangle + \frac{1}{\sqrt{2}}\exp\frac{-i\omega t}{2}|-\rangle\)

The detector detects \(\left||\alpha,0\rangle + |\alpha,t\rangle\right|^2\). Note that the standard hamiltonian phases are equal and so they get washed out, thus we can express path 1 as only the time unevolved state. So, \(I\sim||\alpha,0\rangle + \cdots|^2 = 2+2\Re\langle\alpha,0|\alpha,t\rangle\sim\left(1+\cos\frac{\omega}{2}t\right)\).

\(g_n = -1.91\)

Noether’s Theorem

For every continuous symmetry there exists a corresponding conservation law. I.e. there exists a conserved variable.

Example, for a group, Wigner’s theorem states for a unitary transformation U there is a generator such that \(U=\exp(i\varepsilon G)\) which implies that if \([H,G]=0\) then \(\frac{dG}{dt}=0\). Further, \([G_i=G_j]=c_{ijk}G_k\).

Author: Christian Cunningham

Created: 2024-05-30 Thu 21:20

Validate